2
$\begingroup$

Suppose that $u_k$ is a sequence of $L^1$ functions defined on a compact $K\subset R^n$ and a function $f:[0, \infty)\to[0, \infty)$ with the following properties

  • $u_k\ge 0$
  • $\|u_k\|_{L^1}=\int u_k=1$
  • $u_k\to u$ strongly in $L^1$
  • $f$ is convex, $f(0)=0$ and has superlineair growth at $+\infty$ (that is: $\lim_{z\to+\infty} \frac{f(z)}{z}=+\infty$)
  • $\int f(u_k)< C$ for all $k$

Note that the conditions on $f$ imply that the functional $v\mapsto\int f(v)$ is lower semicontinuous with respect to weak $L^1$ convergence.

Does this imply that $f(u_k)$ converges to $f(u)$ weakly in $L^1$, possibly under the extra assumption that $\int f(u_k)\to \int f(u)$?

EDIT: Thanks for the answers, both were helpful and received an upvote.

$\endgroup$
2
  • $\begingroup$ If you don't assume $\int f(u_k)\to \int f(u)$, you can take as an example $K = [0,1]$, $f(x) = x^2$ and $u_k$ defined as $\sqrt{k}$ on an interval $I_k$ of length $\frac1k$ (so that each $x \in [0,1]$ belongs to infinitely many $I_k$) and as constant otherwise. $\endgroup$ Oct 26, 2012 at 11:47
  • $\begingroup$ And if we assume the extra assumption it is true, and only assuming continuity on f. $\endgroup$ Oct 26, 2012 at 13:58

2 Answers 2

1
$\begingroup$

With the extra assumption it is true, and only continuity on $f:[0,\infty)\rightarrow [0,\infty)$ is needed. Of course, it is sufficient to show that some subsequence of $f(u_k)$ converges. So we can also assume w.l.o.g. that $u_k$ converges a.e. to $u$.

Consider the sequence of non-negative measurable functions on $K$, $v_k:=f\circ u_k$. Because $f$ is continuous, it converges a.e. to $v:=f\circ u$, and by the extra hypothesis, $\int_ K v_k \to\int_K v < \infty $. And, as an immediate consequence of the Fatou's lemma, this also implies $\int_ S v_k \to\int_S v$ for every measurable $S\subset K$ (this is the key point, after all, coming from the equality $\int_K v_k =\int_S v_k +\int_{K\setminus S} v_k $: if one does not lose mass globally, one does not lose mas locally).

Now, as a general fact, on a finite measure space $K$ this situation implies the $L^1$ convergence.

Indeed, let be given a number $\epsilon > 0$. There is a number $\delta > 0 $ such that $\int_ S v < \epsilon$ whenever $S\subset K$ has measure $\big| S\big| < \delta$. By the Severini-Egorov theorem $v_k$ converges almost uniformly, so there is some $S\subset K$ of measure less than $\delta$ such that $v_k$ converges uniformly to $v$ on $K\setminus S$. So we have:

$$ \|v _ k - v\| _ {1,K}=\|v _k-v\| _{1,S}+\|v _k-v\| _{1,K\setminus S}$$ $$\le \int _S v _ k + \int _S v + \big|K\big| \|v _ k-v\| _ {\infty,K\setminus S} \le 2\epsilon + o(1)\, , \quad \mathrm{as}\quad k\to\infty \, .$$

Since this is true for any $\epsilon > 0$ we have $\limsup_{k\to\infty} \|v _ k - v\| _ {1,K}=0$ proving the convergence . Actually, the finiteness assumption on $K$ may also be dropped.

Here's another proof: consider the sequence $ w_k : = \sqrt v_k \in L^2(K)$. It is norm-bounded, converges to $w : = \sqrt{v}$ a.e., hence weakly in $L^2$; moreover, $\|w_ k \| _ 2 \to \|w \| _ 2 $ . In a Hilbert space, this implies strong convergence (this is immediately seen just expanding $\|w _ k - w\| _ 2 ^2$). The map $L^2\ni w\mapsto w^2\in L^1$ is continuous, and we conclude $v_k \to v$ in $L^1$.

$\endgroup$
2
  • $\begingroup$ (the second proof is somehow a little fiddle, since everything is hidden in the fact that: if $w_n$ converges a.e., and converges weakly $L^2$, the two limits coincide a.e. Which is true: but it's actually the point). $\endgroup$ Nov 2, 2012 at 16:55
  • $\begingroup$ Please consider this follow-up question. Thanks in advance $\endgroup$
    – Martijn
    Nov 13, 2012 at 8:51
2
$\begingroup$

I think I can even manage to prove strong convergence in $L^{1}$. I'm going to show that every subsequence has a further subsequence which converges. So, passing to subsequence, we may assume that $u_{k} \to u$ almost everywhere. Since convex functions are continuous (maybe not at $0$)*, we have convergence $f(u_{k}) \to f(u)$ almost evyrewhere. With assumption $\int f(u_{k}) \to \int f(u)$, we can use Scheffe's theorem which states that if a sequence of densities converges almost everywhere to a density, then it is, in fact, strong convergence in $L^{1}$. Of course, we only have $\int f(u_{k}) \to \int f(u)$ instead of $\int f(u_{k}) = \int f(u)$ but it requires no modification of standard proof.

I hope it's quite clear and, what is even more important, doesn't contain any mistake.

*Since $f(0)=0$ and $f$ is nonnegative and convex, then it is also continuous at $0$.

As commented by Pietro Majer, only continuity of $f$ really matters.

$\endgroup$
3
  • 1
    $\begingroup$ See Exercise 4.13 in H.Brezis Functional Analysis, Sobolev Spaces and Partial Differential Equations, Springer Verlag, 2011, p. 121. He leaves plenty of hints on how to solve it. Also, if you assume $f$ convex, you do not need $f$ to be nonnegative. Under the convexity assumption, continuity at $0$ implies continuity everywhere on $[0,\infty)$. $\endgroup$ Oct 26, 2012 at 15:05
  • $\begingroup$ I don't quite understand; is this comment aimed at me or at Martijn? Also, you mean that we can drop nonnegativity when we have convexity but only when we additionally assure continuity at $0$, am I understanding correctly? $\endgroup$ Oct 26, 2012 at 16:33
  • $\begingroup$ Please consider this follow-up question: mathoverflow.net/questions/110741/…. Thanks in advance. $\endgroup$
    – Martijn
    Nov 13, 2012 at 8:52

Your Answer

By clicking “Post Your Answer”, you agree to our terms of service and acknowledge you have read our privacy policy.

Not the answer you're looking for? Browse other questions tagged or ask your own question.